Ayuden con la ecuación
[tex] \times + 15 = 51[/tex]

Answers

Answer 1
X=36 that’s what it gave to me
Answer 2
The answer is:


X= 36
Ayuden Con La Ecuacin[tex] \times + 15 = 51[/tex]

Related Questions

Write the slope-intercept form of the line that has a slope of 2 and intersects the line, 2x - 3y = 6 at x = 3. Include
your work in your final answer. Type your answer in the box provided to submit your solution.

Answers

Answer:

y= 2x -6

Step-by-step explanation:

The slope-intercept form of a line is given by y= mx +c, where m is the slope and c is the y-intercept.

To find the equation of a line, two information are needed:

Slope (given/ calculated)A pair of coordinated

Given that the slope is 2, m= 2. Substitute m= 2 into y= mx +c:

y= 2x +c

Let's find the coordinate in which the line intersects the line 2x -3y= 6. Point of intersection refers to the point at which two lines cuts through each other i.e., the point lies on the graph 2x -3y= 6 and the line of interest.

2x -3y= 6

When x= 3,

2(3) -3y= 6

6- 3y= 6

3y= 6 -6

3y= 0

Divide both sides by 3:

y= 0

Coordinate that lies on the graph is (3, 0).

Substitute the point into the equation and solve for c:

y= 2x +c

When x= 3, y= 0,

0= 2(3) +c

0= 6 +c

c= -6

Substitute the value of c back into the equation:

Thus, the equation of the line in slope-intercept form is y= 2x -6.

Additional:

For a similar question on slope-intercept form, do check out the following!

https://brainly.com/question/28007941

According to the Rational Root Theorem, the following are potential roots of f(x) = 2x2 + 2x – 24.

–4, –3, 2, 3, 4

Which are actual roots of f(x)?
–4 and 3
–4, 2, and 3
–3 and 4
–3, 2, and 4

Answers

The actual roots of f(x) are -4 and 3

How to determine the actual roots?

The function is given as:

f(x) = 2x^2 + 2x – 24.

Expand the function

f(x) = 2x^2 + 8x- 6x – 24.

Factorize the function

f(x) = 2x(x + 4) - 6(x + 4)

Factor out x + 4

f(x) = (x + 4)(2x - 6)

Set to 0

(x + 4)(2x - 6) = 0

Solve for x

x = -4 and 2x = 6

This gives

x = -4 and x = 3

Hence, the actual roots of f(x) are -4 and 3

Read more about actual roots at:

https://brainly.com/question/20896994

#SPJ1

Answer:

A

Step-by-step explanation:

Got 100 on test

Does this appear to be a regular polygon? Explain.

Answers

Answer:

Yes

Step-by-step explanation:

All sides and angles look equal and appears to be a regular hexagon

Hope this helped and have a good day

Answer:

Yes.

Step-by-step explanation:

Hello!

A regular polygon is a closed shape with sides of equal length, and angles of equal degree. A regular polygon also forms around a general center.

This seems to be a regular polygon as all side lengths and angles seem to be equivalent, and there is a center point to the .

This shape is a hexagon, so the measure of the angles are 120°.

If P(En F) = 0.036, P(E|F) = 0.09, and P(F|E) = 0.1, then (a) P(E) = (b) P(F) = = (c) P(EUF) (d) Are the events E and Findependent? =​

Answers

The events E and F are not independent

How to determine the probabilities?

The given parameters are:

P(E n F) = 0.036

P(E|F) = 0.09

P(F|E) = 0.1

To calculate P(E), we use:

P(F|E) = P(E n F)/P(E)

This gives

P(E) = P(E n F)/P(F|E)

So, we have:

P(E) = 0.036/0.1

Evaluate

P(E) = 0.36

To calculate P(F), we use:

P(E|F) = P(E n F)/P(F)

This gives

P(F) = P(E n F)/P(E|F)

So, we have:

P(F) = 0.036/0.09

Evaluate

P(F) = 0.4

To calculate P(E U F), we use

P(E U F) = P(E) + P(F) - P(E n F)

So, we have:

P(E U F) = 0.36 + 0.4 - 0.036

Evaluate

P(E U F) = 0.724

The events E and F are independent if

P(E n F) = P(E) * P(F)

This gives

0.036 = 0.36 * 0.4

Evaluate

0.036 = 0.144 --- false

Hence, the events E and F are not independent

Read more about probabilities at:

https://brainly.com/question/25870256

#SPJ1

In order to pass an exam, a student must answer 70% of the questions correctly. If answering 42 questions correctly results in a 70% score, how many questions are on the test?

Answers

There are 60 questions on the test

Calculating percentages

Total number of questions = 42

Percentage equivalent= 70%

Let the total number of questions in the test be represented by x

42 = 70% of x

[tex]42=\frac{70}{100} \times x[/tex]

42 = 0.7x

Divide both sides by 0.7

42/0.7  =  0.7x/0.7

x  =  60

Therefore, there are 60 questions on the test

Learn more on percentages here: https://brainly.com/question/11360390

#SPJ1

A zoo has a black rhinoceros that weighs 18 times as much as an average-size chimpanzee. The rhinoceros weighs 2,250 pounds.

How much does an average-size chimpanzee weigh?

Enter the answer in the box. [_]

Answers

Answer: 125 pounds

Step-by-step explanation:

Let's first represent the weight of the black rhinoceros and chimpanzee with variables. We will assign r for the rhinoceros and c for the chimpanzee.

In the question, we were given that the weight for a black rhinoceros is 18 times as much as an average-size chimpanzee. This makes 18 times the chimpanzee's weight equal to the rhinoceros' weight.

Let's put this into an equation.

[tex]r=18*c[/tex]

Now, let's put in the values we know to find the weight of the chimpanzee (i.e., c). We know the weight of a rhinoceros (represented by r) is 2250 pounds, so we can replace it for r in the equation.

[tex]2250=18*c[/tex]

To solve this equation, we can divide both sides by 18 to get c by itself and know what it's value is.

[tex]\frac{2250}{18}=\frac{18*c}{18}[/tex]

We can remove the [tex]\frac{18}{18}[/tex] from the right side to be left with c. We can also divide 2250 by 8 to get the weight of an average-size chimpanzee.

[tex]c=\frac{2250}{18}\\ c=125[/tex]

Hence, an average-size chimpanzee weighs 125 pounds.

Select the two values of x that are roots of this equation x^2-5x+2=0

Answers

The roots of the equation is x = 4.56 OR x = 0.44

Quadratic equation

From the question, we are to determine the roots of the given equation

The given equation is

x² -5x +2 = 0

Using the formula method,

[tex]x =\frac{-b \pm \sqrt{b^{2}-4ac } }{2a}[/tex]

In the given equation,

a = 1, b = -5, c = 2

Putting the values into the formula,

[tex]x =\frac{-(-5) \pm \sqrt{(-5)^{2}-4(1)(2) } }{2(1)}[/tex]

[tex]x =\frac{5 \pm \sqrt{25-8} }{2}[/tex]

[tex]x =\frac{5 \pm \sqrt{17} }{2}[/tex]

[tex]x =\frac{5 + \sqrt{17} }{2}[/tex] OR [tex]x =\frac{5 - \sqrt{17} }{2}[/tex]

[tex]x =\frac{5 + 4.12}{2}[/tex] OR [tex]x =\frac{5 - 4.12}{2}[/tex]

[tex]x =\frac{9.12}{2}[/tex] OR [tex]x =\frac{0.88}{2}[/tex]

x = 4.56 OR x = 0.44

Hence, the roots of the equation is x = 4.56 OR x = 0.44

Learn more on Quadratic equation here: https://brainly.com/question/8649555

#SPJ1

Jackson invested $4,200 in an account paying an interest rate of 9 1/2 compounded continuously. Julia invested $4,200 in an account paying an interest rate of 8 7/8 compounded quarterly. To the nearest hundredth of a year, how much longer would it take for Julia's money to double than for Jackson's money to double?

Answers

It would Julia 0.60 years more to double the initial investment.

What is future value?

Future value means the initial investment multiplied by 2 since the future value is meant to double.

The formula for future value of a continuously interest rate is provided below:

FV=PV*e^(rt)

FV=future value=$4,200*2=$8,400

PV=initial investment=$4,200

e=exponential constant=2.7182818

r=interest rate=9.5%

t=number of years it takes for the investment to double=unknown

$8,400=$4,200*2.7182818^(9.5%*t)

$8,400/$4,200=2.7182818^(0.095t)

2=2.7182818^0.095t

take log of both sides

ln(2)=0.095t* ln(2.7182818)

0.095t=ln(2)/ln(2.7182818)

0.095t=0.69314718781684800

t=0.69314718781684800/0.095

t=7.30 years

The future value when interest is compounded quarterly is shown thus:

FV=PV*(1+r/4)^(N*4)

FV=$8,400

PV=$4,200

r=8 7/8%

r=8.875%

N=the number of years it would take for the initial investment to double=unknown

$8,400=$4,200*(1+8.875%/4)^(N4)

$8,400=$4,200*(1+0.0221875)^(N4)

$8,400/$4,200=(1+0.0221875)^(N4)

2=(1+0.0221875)^(N4)

2=(1.0221875)^(N4)

take log of both sides

ln(2)=N4*ln(1.0221875)

N4=ln(2)/ln(1.0221875)

N4=31.5857423180125

N=31.5857423180125/4

N=7.90

Difference in years=7.90-7.30

difference in years=0.60 years

Find more on continuously compounded interest formula below:

https://brainly.com/question/3932907

#SPJ1

Find the first 10 terms of the sequence below :
g) the sequence whose terms are constructed sequen tially as follows: start with 1, then add 1, then mul
tiply by 1, then add 2, then multiply by 2, and so on
h) the sequence whose nth term is the largest integer k
such that k!

Answers

The first ten terms of the sequence are 1, 2, 8, 33, 148, 765, 4626, 32431, 259512, 2335689.

The n-th term of the sequence is aₙ ₊ ₁ = (aₙ + 1) · n.

How to generate the elements of a sequence

A sequence is a set of elements generated by at least one condition, usually an equation. In this case, the sequence is generated by a recurrence formula:

a₁ = 1, aₙ ₊ ₁ = (aₙ + 1) · n        (1)

The first ten terms of the sequence are:

n = 1

a₂ = (a₁ + 1) · 1

a₂ = 2

n = 2

a₃ = (a₂ + 2) · 2

a₃ = 4 · 2

a₃ = 8

n = 3

a₄ = (a₃ + 3) · 3

a₄ = 11 · 3

a₄ = 33

n = 4

a₅ = (a₄ + 4) · 4

a₅ = 37 · 4

a₅ = 148

n = 5

a₆ = (a₅ + 5) · 5

a₆ = 153 · 5

a₆ = 765

n = 6

a₇ = (a₆ + 6) · 6

a₇ = (765 + 6) · 6

a₇ = 4626

n = 7

a₈ = (a₇ + 7) · 7

a₈ = 4633 · 7

a₈ = 32431

n = 8

a₉ = (a₈ + 8) · 8

a₉ = 32439 · 8

a₉ = 259512

n = 9

a₁₀ = (a₁₀ + 9) · 9

a₁₀ = 259521 · 9

a₁₀ = 2335689

To learn more on sequences: https://brainly.com/question/21961097

#SPJ1

4x + 4y = 40
2x - 4y = 8

Answers

Answer:

x=8   y=2

Step-by-step explanation:

solve for x

1. 4x+4y=40

2. subtract 4y from both sides

3. 4x=40-4y

4. divide both sides by 40

5. [tex]\frac{4x}{4} = \frac{40-4y}{4}[/tex]

6. dividing by four undoes the multiplication by four

7.    [tex]x=\frac{40-4y}{4}[/tex]

8. divide 40 - 4y by 4

9. x=10-y

10. use the last equation to solve the rest

An equation is shown below: 8x + 2(x – 7) = 7x + 3x – 14 Part A: Solve the equation and write the number of solutions. Show all the steps. (6 points) Part B: Name one property you used to solve this equation. (4 points) Source StylesNormalFontSize

Answers

Answer:

Infinitely ManyDistributive Property

Step-by-step explanation:

8x + 2(x - 7) = 7x + 3x - 14

8x + 2x - 14 = 7x + 3x - 14                 Distributive property.

10x - 14 = 10x - 14                              Combine the like terms.

-14 = -14                                             Subtraction.

0 = 0                                                  Addition.

Since the statement 0 = 0 is true regardless of the value of x, there is infinitely many solutions.

A wire is stretched from the ground to the top of an antenna tower. The wire is 15 feet long. The height of the tower is 3 feet greater than the distance d from the tower's base to the end of the wire. Find the distance d and the height of the tower.

Answers

The distance d is 9 ft and the height is 12ft.

How to find the distance and the height?

Here we can model the situation with a right triangle, where the length of the wire is the hypotenuse.

The height is one cathetus and the distance is the other catheti.

Let's define:

h = heightd = distance.hypotenuse = 15ft

We know that the height of the tower is 3 ft larger than the distance, then:

h = d + 3ft

Now we can use the Pythagorean theorem, it says that the sum of the squares of the cathetus is equal to the square of the hypotenuse.

Then:

[tex]d^2 + (d + 3ft)^2 = (15ft)^2[/tex]

Now we can solve this equation for d:

[tex]d^2 + d^2 + 6ft*d + 9ft^2 = (15ft)^2\\\\2d^2 + 6ft*d - 216 ft^2 = 0\\\\d^2 + 3ft*d - 108ft^2 = 0[/tex]

Then the solutions are:

[tex]d = \frac{-3ft \pm \sqrt{(3ft)^2 - 4*(-108ft^2)} }{2} \\\\d = \frac{-3ft \pm 21ft }{2}[/tex]

We only take the positive solution:

d = (-3ft + 21ft)/2 = 9ft

And the height is 3 ft more than that, so:

h = 9ft + 3ft = 12ft

The distance d is 9 ft and the height is 12ft.

If you want to learn more about right triangles:

https://brainly.com/question/2217700

#SPJ1

1. There are 50 contestants signed up for a TV show. There are 36 more female contestants than male contestants. How many female contestants have signed up to compete? Show your solution and explain how you plan to explain this to your students.

Answers

Answer:

males = 7

females = 43

Step-by-step explanation:

whilst it may seem intuitive to simply subtract 36 from 50, it is not saying "there are 36 males, how many females?" but instead, "the difference between the number of males and females is 36".

You can solve this equation most easily algebraically. For example:

Number of males = x

number of females = y

the question states that the total number of people = 50

therefore we can say that the total number of males (x) + the total number of females (y) = 50 people

therefore: x + y = 50

similarly, the question says that the number of males (x) + 36 = the total number of females (y)

therefore: x + 36 = y

we now have two equations:

x + y = 50

x + 36 = y

whilst both equations have two unknowns (x and y), therefore we can't simple solve for x or y, with the combination, we can see a pattern.

focusing on the second equation: x + 36 = y

we can add x to both sides, because you can pretty much do anything to the equation as long as you do it to both sides.

x + 36 + x = y + x

now this may seem very random, but you now see that one side of the equation equals y + x, and remember from the other equation, x + y = 50. Therefore we can substitute x + y in the second equation for 50.

our two equations:

x + 36 + x = y + x

x + y = 50

therefore:

x + 36 + x = 50

for the sake of clarity, we can combine like terms...

x + x = 2x

therefore:

x + 36 + x = 50

2x + 36 = 50

solve for x by subtracting 36 from both sides, then dividing both sides by 2

2x + 36 - 36 = 50 - 36

2x = 14

2x / 2 = 14 / 2

x = 7

now remember:

Number of males = 7 (we now know x = 7)

now that we've solved for x, we can go back to our original equation:

x + 36 = y

and substitute x...

7 + 36 = y

43 = y

Now remember:

Number of females = 43 (we now know y = 43)

therefore there are 7 males and 43 females. we can proof this by adding 7 and 43, and you'll see you reach 50, which is the correct total number of people.

hope this helps :)

lim x→-1 x^m + 1/x^n + 1

Answers

I assume [tex]m,n[/tex] are integers to avoid (ir)rational powers of -1.

If [tex]m,n[/tex] are both even, or if [tex]m=n[/tex], then

[tex]\displaystyle \lim_{n\to-1} \frac{x^m+1}{x^n+1} = \frac{1+1}{1+1} = 1[/tex]

If [tex]m,n[/tex] are both odd and [tex]m\neq n[/tex], then we can factorize

[tex]\dfrac{x^m+1}{x^n+1} = \dfrac{(x+1)(x^{m-1} - x^{m-2} + \cdots - x + 1)}{(x+1)(x^{n-1}-x^{n-2}+\cdots-x+1)}[/tex]

Note that there are [tex]m[/tex] terms in the numerator and [tex]n[/tex] terms in the denominator.

In the limit, the factors of [tex]x+1[/tex] cancel and

[tex]\displaystyle \lim_{x\to-1} \frac{x^m+1}{x^n+1} = \lim_{x\to-1} \frac{x^{m-1} - x^{m-2} + \cdots - x + 1}{x^{n-1}-x^{n-2}+\cdots-x+1} \\\\ ~~~~~~~~~~~~~~~~~~= \dfrac{1-(-1)+1-(-1)+\cdots-(-1)+1}{1-(-1)+1-(-1)+\cdots-(-1)+1} \\\\ ~~~~~~~~~~~~~~~~~~=\frac{1+1+\cdots+1}{1+1+\cdots+1} = \dfrac mn[/tex]

If [tex]m[/tex] is even and [tex]n[/tex] is odd, then we can only factorize the denominator and the discontinuity at [tex]x=-1[/tex] is nonremovable, so

[tex]\displaystyle \lim_{x\to-1}\frac{x^m+1}{x^n+1} = \lim_{x\to-1} \frac{x^m+1}{(x+1)(x^{n-1}-x^{n-2}+\cdots-x+1)} \\\\ ~~~~~~~~~~~~~~~~~~= \frac2m \lim_{x\to-1} \frac1{x+1}[/tex]

which does not exist.

If [tex]m[/tex] is odd and [tex]n[/tex] is even, then we can factorize the numerator so that

[tex]\displaystyle \lim_{x\to-1}\frac{x^m+1}{x^n+1} = \lim_{x\to-1} \frac{(x+1)(x^{m-1}-x^{m-2} +\cdots -x+1)}{x^n+1} \\\\ ~~~~~~~~~~~~~~~~~~= \frac{0m}2 = 0[/tex]

Jeremiah is a salesperson who sells computers at an electronics store. He makes a base pay amount each day and then is paid a commission as a percentage of the total dollar amount the company makes from his sales that day. Let PP represent Jeremiah's total pay on a day on which he sells xx dollars worth of computers. The table below has select values showing the linear relationship between xx and P.P. Determine how many dollars worth of computers Jeremiah would have to sell in order to get paid $130 on a given day.

Answers

Jeremiah has to sell 5000 dollars worth of computers to get paid $130 on a given day. Using the linear equation, the required value is calculated.

What is a linear equation?

An equation in which if the highest degree of the variable is 1(one), then that equation is said to be a linear equation.

General form: ax + b = c; where the power of the variable x is 1.

Calculation:

It is given that,

Jeremiah makes a base pay amount each day and then is paid a commission as a percentage of the total dollar amount the company makes from his sales that day.

Consider,

P - as total pay on a day, x - as the number of dollars worth of computers, B - as basic pay, and C - as commission percentage.

So, the linear equation that relates x and P is,

P = Cx + B   ...(i)

On substituting the values from the given table we get,

122.5 = C(4500) + B ...(ii)

160 = C(7000) + B ...(iii)

175 = C(8000) + B ...(iv)

By solving equations (iii) and (iv), we get

C = 15/1000 = 0.015

B = 55

Finding x value when P = $130:

We have P = Cx + B. Then for P = 130,

130 = Cx + B

We know C = 0.015 and B = 55

On substituting these values,

130 = (0.015) x + 55

⇒ 0.015x = 130 - 55 = 75

x = 75/0.015 = 5000

Therefore, the required computers are 5000 dollars worth.

Learn more about linear equations here:

https://brainly.com/question/2030026

#SPJ1

Disclaimer: The given question on the portal was incomplete. Here is the complete question.

Question: Jeremiah is a salesperson who sells computers at an electronics store. He makes a base pay amount each day and then is paid a commission as a percentage of the total dollar amount the company makes from his sales that day. Let P represent Jeremiah's total payments on a day on which he sells x dollars worth of computers. The table below has select values showing the linear relationship between x and P. Determine how many dollars worth of computers Jeremiah would have to sell to get paid $130 on a given day.

Table:

x: 4500, 7000, 8000

P: 122.5, 160, 175

respectively.

Use the diagram to determine which statement is true

Answers

The answer is d.

Finding area of ABCD :

Find side lengthside = √3² + 4²side = 5

    2. Apply formula to find area

area = 5²area = 25

Finding area of GHIA :

area = 4²area - 16

Finding area of DEFG :

area = 3²area = 9

Now, let's see whether is true.

Area (ABCD) - Area (GHIA) = Area (DEFG)25 - 16 = 99 = 9

∴ Hence, it is proved √

Solve for v. -8=-2/v

Answers

Answer is 1/4
Step by step
-8 = -2/v
First we want to isolate “v”
-2 is divided by “v”, so we will multiply both sides by “v”
-8v = -2
Now we can solve for v
Divide both sides by -8
v = 2/8, simplified to 1/4
(Remember when you multiply or divide two negatives it becomes a positive)

If you want to check your work, substitute your answer of 1/4 into the original equation.

-8= (-2) divided by (1/4)
-8 = -8
Problem solved!!

Can you please help me with this?

Answers

the answer to your question is Y=0

18. Sam retires in 1996. He has an amount of R350 000 available to invest. He decides to buy a second house for 50% of the money, which he lets at an amount of R2000 per month. He increases the rent every year by an amount of R300. The balance of R175 000 he invests in the bank at a rate of 12%. He uses the interest every month to supplement his income, so the interest is not compounded. He also gets a pension of R3000 per month, which is increased by R300 per month every year. What was his monthly income in 1996? (1)​

Answers

If Sam had Rs. 350000 and invested Rs.175000 in house, in bank Rs.175000 and getting 3000 pension then the monthly income was Rs. 6750.

Given that Sam had Rs.350000,investment in house Rs.175000 at a rent of Rs.2000 per month and Rs.175000 in bank at rate of 12%, getting pension of Rs.3000 per month.

We are required to find the monthly income in 1996.

We have assumed that Sam was retired on 1st January, 1996 so the amount of rent, investment in bank and pension did not increase because they had to be increase in a year and we have to calculate the monthly income in which he was retired.

Monthly income=Rent of 1 month+Simple interest of 1 month+Pension per month

=2000+175000*1/100+3000

=2000+1750+3000

=Rs.6750

Hence if Sam had Rs. 350000 and invested Rs.175000 in house, in bank Rs.175000 and getting 3000 pension then the monthly income was Rs. 6750.

Learn more about simple interest at https://brainly.com/question/25793394

#SPJ1

20 pts and brainliest

Answers

The two solutions to the given quadratic equation are 2i/3, -2i/3 and they are both complex solutions.

Hence, option C is the correct answer.

What are the solutions to the quadratic equation?

Given the equation; 9x² + 4 = 0

First, we subtract 4 from both sides.

9x² + 4 = 0

9x² = -4

x² = -4/9

Take the square roots of both sides

x = ±√(-4/9)

Rewrite -4/9 as (2i/3)²

x = ±√(2i/3)²

x = ±(2i/3)

Hence,

x = 2i/3, -2i/3

Therefore the two solutions to the given quadratic equation are 2i/3, -2i/3 and they are both complex solutions.

Hence, option C is the correct answer.

Learn more about quadratic equations here: brainly.com/question/1863222

#SPJ1

Answer:

but it says ill get 10 only

Step-by-step explanation:

Consider the diagram shown and answer the following questions; the radius of this circle is 6 inches.

a. Define how lines a, b, c, and d relate to circle P. (What special names do these lines have in relation to the circle?)
b. If the measure of angle OPS is 139°, what extra information would we need to calculate the measure of angle ORS using intersecting chords? Explain how we can use this information to calculate the angle.
c. Segment NS is 14 the length of segment TO. Explain how theorem 65 would allow us to calculate the length of segments RO, RS, RV, and RT.

Answers

The additional information needed to calculate ORS are the measures of SPR and PSR

The special names of the lines

The lines are given as:

Lines a, b, c and d

The special names of the lines are as follows:

Line a: A secant. This is because the line divides the circle into unequal segmentsLine b: A tangent: This is because the line touches the circle at a point on the circumferenceLine c: A diameter. This is because the line divides the circle into equal segmentsLine d: A secant. This is because the line divides the circle into unequal segments

The additional information needed

The angle is given as:

OPS= 139 degrees

Start by calculating SPR using

SPR = 180- 139

SPR = 41 degrees

So, the additional information needed to calculate ORS are the measures of SPR and PSR

How to calculate the lengths RO, RS, RV, and RT

The theorem 65 is not stated.

So, the question cannot be answered

Read more about circle theorems at:

https://brainly.com/question/19906313

#SPJ1

Find the integrals:
∫30x^2/√(x-4) dx
u=x-4 and u=√(x-4)

Answers

I assume you're asked to compute

[tex]\displaystyle \int \frac{30x^2}{\sqrt{x-4}} \, dx[/tex]

using both of the substitutions provided.

With [tex]u=x-4[/tex], we have [tex]x=u+4[/tex] and [tex]dx=du[/tex]. Then

[tex]\displaystyle \int \frac{30x^2}{\sqrt{x-4}} \, dx = \int \frac{30(u+4)^2}{\sqrt u} \, du \\\\ ~~~~~~~~ = 30 \int \frac{u^2 + 8u + 16}{\sqrt u} \, du \\\\ ~~~~~~~~ = 30 \int \left(u^{3/2} + 8u^{1/2} + 16u^{-1/2}\right) \, du \\\\ ~~~~~~~~ = 30 \left(\frac25 u^{5/2} + \frac{16}3 u^{3/2} + 32 u^{1/2}\right) + C \\\\ ~~~~~~~~ = 12 u^{5/2} + 160 u^{3/2} + 960 u^{1/2} + C \\\\ ~~~~~~~~ = 12 (x-4)^{5/2} + 160 (x-4)^{3/2} + 960 (x-4)^{1/2} + C \\\\ ~~~~~~~~ = 4 \sqrt{x-4} \left(3 (x-4)^2 + 40 (x-4) + 240\right) + C \\\\ ~~~~~~~~ = \boxed{4 \sqrt{x-4} \left(3x^2 + 16x + 128\right) + C}[/tex]

With [tex]u=\sqrt{x-4}[/tex], we have

[tex]u^2 = x-4 \implies x^2 = (u^2+4)^2[/tex]

and [tex]2u\,du=dx[/tex]. Then

[tex]\displaystyle \int \frac{30x^2}{\sqrt{x-4}} \, dx = \int \frac{60u \left(u^2+4\right)^2}u \, du \\\\ ~~~~~~~~ = 60 \int \left(u^4 + 8u^2 + 16\right) \, du  \\\\ ~~~~~~~~ = 60 \left(\frac15 u^5 + \frac83 u^3 + 16u\right) + C  \\\\ ~~~~~~~~ = 12 (x-4)^{5/2} + 160 (x-4)^{3/2} + 960 (x-4)^{1/2} + C \\\\ ~~~~~~~~ = 4 \sqrt{x-4} \left(3 (x-4)^2 + 40 (x-4) + 240\right) + C \\\\ ~~~~~~~~ = \boxed{4\sqrt{x-4} \left(3x^2 + 16x + 128\right) + C}[/tex]

Which figures demonstrate a translation?

Answers

The two bottom graphs demonstrate translations.

Which figures demonstrate a translation?

We will have a translation only if:

The size of the figure does not change (like in option 1, which we can discard).If the "direction" of the figure does not change, like in option 2, where you can see that there is a reflection.

The images where the figures are only moved a little bit are the ones that demonstrate just a translation, and these are the two lower ones.

If you want to learn more about translations:

https://brainly.com/question/24850937

#SPJ1

Ivanhoe Company is considering buying a new farm that it plans to operate for 10 years. The farm will require an initial investment of $11.85 million. This investment will consist of $2.15 million for land and $9.70 million for trucks and other equipment. The land, all trucks, and all other equipment are expected to be sold at the end of 10 years for a price of $5.25 million, which is $2.00 million above book value. The farm is expected to produce revenue of $2.10 million each year, and annual cash flow from operations equals $1.90 million. The marginal tax rate is 35 percent, and the appropriate discount rate is 10 percent. Calculate the NPV of this investment. (Do not round factor values. Round final answer to 2 decimal places, e.g. 15.25.)

Answers

The  NPV of this investment if the discount rate is 10 percent is: 1.58%.

Net present value (NPV)

Year Cash flow  PVIF 10%  Present value

0         ($11.86)     1.000           ($11.86)

1           1.90          0.909           $1.73

2          1.90          0.826           $1.57

3          1.90           0.751           $1.43

4          1.90           0.683          $1.30

5          1.90           0.621           $1.18

6          1.90           0.564          $1.07

7          1.90            0.513          $0.98

8          1.90           0.467          $0.89

9          1.90           0.424          $0.81

10         6.45          0.386         $2.49

NPV                                          $1.58

1.9+5.25-2×35%=6.45

Hence, the NPV is $1.58.

Learn more about NPV here:https://brainly.com/question/17185385

#SPJ1

The graph of f(x) = x3 − 7x − 6 is shown.



Based on the graph, what are all of the solutions to f(x) = x3 − 7x − 6?

x = −6
x = −2, −1
x = −2, −1, 3
x = −6, −2, −1, 3

Answers

In accordance with the graph of the cubic function, the roots are - 2, - 1, 3.

What are the roots of a cubic equation according to a graph?

In this question we have a graph of a cubic equation, the roots are the points of the curve that pass through the x-axis. Cubic equations have at least a real root and at most three. In accordance with the graph of the cubic equation, the roots are - 2, - 1, 3.

To learn more on cubic equations: https://brainly.com/question/13730904

#SPJ1

Question
n⃗ =⟨−2, −1⟩ and D=[−4423].

What is D⋅n⃗ ?

Enter your answer as a vector by filling in the boxes.

Answers

The dot product of the D⋅n is 32.

According to the statement

We have given the value of n vector and d matrix and we have to find the dot product of these.

So, For this purpose,

The given values:

n = {-2,-1} and D = [−4423].

The dot product or scalar product is an algebraic operation that takes two equal-length sequences of numbers, and returns a single number.

So, The d matrix become

[tex]D = \left[\begin{array}{cc}-4&4&\\2&3&\\\end{array}\right][/tex]

Now solve it with the help of multiplication then the matrix become

D = (-12, -8)

and n = {-2,-1}

Now multiply both terms with the dot product.

So, the dot product of the both terms will become

D.n = 24 +8

Then

The output of the dot product of both terms is 32.

So, The dot product of the D⋅n is 32.

Learn more about Dot product here

https://brainly.com/question/2289103

#SPJ1

help!! How do I solve for x and what is x

Answers

Answer:

x=75 degrees

Step-by-step explanation:

since the shape is quadrilateral, all the angles added together should equal 360 degrees so you use 360 to subtract all the given angles on the shape and you can find X

360-131-107-47=75

A Ferris wheel with a diameter of 10 m and makes one complete revolution every 80
seconds. Assume that at time t = 0, the Ferris Wheel is at its lowest height above
the ground of 2 m. You will develop the equation of a cosine graph that models your
height, in metres, above the ground as you travel on the Ferris Wheel over time, t in
seconds. To do this, answer the following questions.


1. State the amplitude of the graph.
2. State the value of k in the general form y = a cos [k(x − d)] + c.
-
3. State the value of d.
4. State the value of c.
5. State the cosine equation of the graph.

Answers

A Ferris wheel with a diameter of 10 m and makes one complete revolution every 80 seconds. Assuming that at time t = 0, the Ferris Wheel is at its lowest height above the ground of 2 m, the cosine equation of the graph drawn is, y = 5 cos [( π/40)(x - (π/2))] + 3.  Here, amplitude of the graph is 5, value of k is  π/40, d is π/2 and c is 3.

Developing the Equation of a Cosine Graph

The given information constitutes the following,

Diameter = 10 m

⇒ Radius, r = 5 m

Time, t = 80 s

Height above the ground, h = 2 m

Thus, we can infer that,

Amplitude, A = 5 m

Period, T = 80 s

Minimum height = 2 m

The cosine function is given as,

a cos [k(x − d)] + c

Here, A is amplitude

B is cycles from 0 to 2π and thus period = 2π/k

d is horizontal shift

c is vertical shift (displacement)

Now, 2π/k = 80

⇒ k = 2π/80 = π/40

The value of c is given as,

c = Amplitude - Minimum height

c = 5 - 2

c = 3

For a shift to the left by π/2 gives, we have,

d = π/2

Thus, the desired equation of the drawn cosine graph is,

y = 5 cos [( π/40)(x - (π/2))] + 3

Learn more about amplitude here:

https://brainly.com/question/8662436

#SPJ1

Help me please I’m not the smartest

Answers

x^2=16
x= 4 OR x=-4

The picture attached below is your answer because it’s between 4 and -4.

!!!!!!!!!!!!!!!!!!!!!!helpppppppo

Answers

Answer:

(3x3+3)*2

Step-by-step explanation:

Other Questions
the unique material formed from dense packing of neurons and glia in the cerebral cortex is called: ____ calculate the age of the universe using each galaxy and then average them. there are seven data points, so add them all together and divide by 7. How would you find the actual amount of milligrams of Mg(OH)2 present in 5mL of milk of magnesia? 4. Microtubules are inherently unstable unless they areSelect one:a. able to bind GTP.b. stabilized by a (+) end attachmentc. bound to kinesind. assembled in nucleus The wavelength of the red light from a calcium flame is 617 nm. This light originated from a calcium atom in the hot flame. In the calcium atom from which this light originated, what was the period of the simple harmonic motion which was the source of this electromagnetic wave? determine central ideas why was it difficult for non-jewish civilians to help, hide, or protect their jewish neighbors? which phase of the uterine cycle is characterized by regrowth of the functional layer of the endometrium? Thomas has a classic 1966 Ford Mustang, and Jeffrey would like to buy it. The Mustang has less than ten thousand (10,000) original miles, it is in pristine condition, and it was only the third Mustang to "roll off the Ford assembly line in Dearborn, Michigan in late 1965. Thomas and Jeff actually enter into a written contract for Jeff's purchase of the car for $75,000, but Thomas later backs out of the deal for sentimental reasons (his late mother gave Thomas the car for his sixteenth birthday.) Jeff has already paid Thomas the $75,000 purchase price. In a breach of contract action against Thomas, which of the following is the likely remedy that Jeff will request and that the court will likely award him? Multiple Choice a. Substantial performance b. Specific performance c. Substantial performance and $75,000 d. Specific performance and $75,000 e. Substantial performance and $300,000 (representing $75,000 in compensatory damages and $225,000 in statutory punitive damages) if the wage is temporarily decreased to $15, it will need to ________ to restore equilibrium. Match the lock type in multigranular locking to its associated action.1. Shared lock (S)2. Exclusive lock (X)3. Intention shared (IS)4. Intention exclusive (IX)5. Shared intention exclusive (SIX)a) Used when writing an object is required.b) Logically equivalent to holding both a shared(S) and intention exclusive (IX) lock.c) Used when only reading an object is required.d) Used to lock the ancestors of a object being locked in shared mode.e) Used to lock the ancestors of an object being locked in exclusive mode. Calculate [oh] in a solution obtained by adding 1. 50 g solid koh to 1. 00 l of 10. 0 m nh. (kb of nh is 1. 80 10) a resistor with r1 = 29.0 is connected to a battery that has negligible internal resistance and electrical energy is dissipated by at a rate of 50.0 w..If a second resistor with R2 = 15 is connected in series with R1, what is the total rate at which electrical energy is dissipated by the two resistors? Express your answer using two significant figures. Given the code below:def frequency (itemList): counters { } for item in itemList: if = ___ ___ = 1 else: ____ += 1 return counters grades [95, 96, 100, 85, 95, 90, 95, 100, 100] print (frequency (grades) ) Fill in the blanks in the code above so that the resulting output looks as follows:{95: 3, 96: 1, 100: 3, 85: 1, 90: 1} As you work on constructing the price for a Voluntrip tour, consider these guiding questions: What does the product/service cost? What price will the market bear? What pricing strategy/method should be used? How will the price compare to that of the competitors? Where are these consumers located? Do these consumers have specific interests or needs? Calculate the energy released when 100.0 g of steam at 110.0 c are converted into ice at minus 30.0 c charged with the responsibility to __________, the __________ is almost certainly the most powerful committee in either chamber of congress. Sparta Landscaping Company established the following standard labor cost data to provide complete lawn care service (cutting, edging, trimming, and blowing) for a small lawn. Sparta planned each lawrn to require 2 hours of labor at a cost of $18.00 per hour. The company actually serviced 500 lawns us- ing an average of 1.75 labor hours per lawn. Actual labor costs were $19.20 per hour Required a. Determine the total labor variance and indicate whether the variance is favorable (F) or b. Determine the labor price variance and indicate whether the variance is favorable (F) or c. Determine the labor usage variance and indicate whether the variance is favorable (F) or d. Explain what could have caused the variances computed in Requirements b and c. unfavorable (U). unfavorable (U). unfavorable (U). May the vendor of an installment land contract containing a forfeiture clause reinstate strict performance once she has waived it?A. Yes, by establishing a pattern of accepting on-time paymentsB. Yes, by giving the purchaser notice and a reasonable time to catch up on paymentsC. No; a vendor's waiver is permanent Claim: The mean systolic blood pressure of women aged 40-50 in the U.S. is equal to 126 mmHg.Test statistic: z = 1.72A)0.9146B)0.0472C)0.9573D)0.0854 How many hydroxide ions are present in 10grams of Barium hydroxide